Download Full Topic PDF

Survey
yes no Was this document useful for you?
   Thank you for your participation!

* Your assessment is very important for improving the workof artificial intelligence, which forms the content of this project

Document related concepts

Medicine wikipedia , lookup

Patient safety wikipedia , lookup

Medical ethics wikipedia , lookup

Adherence (medicine) wikipedia , lookup

Epidemiology of snakebites wikipedia , lookup

Transcript
Diagnosis And Management Of
North American Snake And
Scorpion Envenomations
Case #1: A 45-year-old man presents, via rural EMS, with a chief complaint
of “snakebite.” EMS reports that the patient ran over a 6 to 8ft snake with his
pickup truck, cutting it in half. The patient doubled back to pick up the dead
snake (foreseeing a nice new pair of snakeskin boots) and when he grabbed it,
the snake “came around” and bit his hand. He has two puncture wounds almost
7cm apart on his right hand surrounded by dark purple ecchymosis and the start
of bullae formation. He has edema and erythema of his hand and arm to the
elbow and is having fasciculations of most of his large skeletal muscle groups. He
is in pain and EMS reports that he seemed to be getting progressively confused
and lethargic during their 70 minute transport. He is hypotensive, tachycardic,
and is oozing blood from his IV sites. You realize that his life depends on your
actions...
Case #2: A 30-year-old man presents with a chief complaint of “snakebite.”
About 30 minutes prior to arrival he was wading in murky water cleaning debris
out of a stream and felt a bite on his hand. As he pulled his hand out of the water,
he briefly caught a glimpse of a snake as it swam away; he gives a fairly good
description of a small copperhead. He has two small punctures on his right index
finger about 1 cm apart. He is currently pain free, without erythema, swelling, or
ecchymosis. His vitals and remaining physical exam are normal. He wants to
know if he can go home...
Case #3: A 39-year-old man presents to your Florida ED about four hours
after being bitten on the forearm by a small red, yellow, and black snake while
clearing brush. The snake escaped and was thought to be a king snake. There was
minimal pain at the site of the bite and no swelling or bleeding, so he continued
working. The patient started to get worried when he developed twitching in his
arms and face about one hour before arrival at the ED. Soon after the twitching
started, he began having difficulty talking and swallowing and had an episode of
double vision, prompting him to call 911. On arrival, the patient is awake, alert,
Editor-in-Chief
Andy Jagoda, MD, FACEP, Professor
and Vice-Chair of Academic Affairs,
Department of Emergency Medicine;
Mount Sinai School of Medicine;
Medical Director, Mount Sinai Hospital,
New York, NY.
Associate Editor
Health Science Center, New Orleans,
LA.
Wyatt W Decker, MD, Chair and
Associate Professor of Emergency
Medicine, Mayo Clinic College of
Medicine, Rochester, MN.
Francis M Fesmire, MD, FACEP,
Director, Heart-Stroke Center,
Erlanger Medical Center; Assistant
Professor, UT College of Medicine,
Chattanooga, TN.
HSC/Jacksonville, FL.
Gregory L Henry, MD, FACEP, CEO,
Medical Practice Risk Assessment,
Inc; Clinical Professor of Emergency
Medicine, University of Michigan, Ann
Arbor.
Keith A Marill, MD, Instructor,
Department of Emergency Medicine,
Massachusetts General Hospital,
Harvard Medical School, Boston, MA.
John M Howell, MD, FACEP, Clinical
Charles V Pollack, Jr, MA, MD, FACEP,
Professor of Emergency Medicine,
Michael J Gerardi, MD, FAAP, FACEP,
Professor and Chair, Department of
George Washington University,
Director, Pediatric Emergency
Emergency Medicine, Pennsylvania
Washington, DC; Director of Academic
Medicine, Children’s Medical Center,
Hospital, University of Pennsylvania
Affairs, Best Practices, Inc, Inova
Atlantic Health System; Department of
Health System, Philadelphia, PA.
Fairfax Hospital, Falls Church, VA.
Emergency Medicine, Morristown
Memorial Hospital, NJ.
Michael S Radeos, MD, MPH,
Editorial Board
Assistant Professor of Emergency
Michael A Gibbs, MD, FACEP, Chief,
Medicine, Lincoln Health Center,
William J Brady, MD, Associate
Department of Emergency Medicine,
Bronx, NY.
Professor and Vice Chair, Department
Maine Medical Center, Portland, ME.
of Emergency Medicine, University of
Robert L Rogers, MD, FAAEM,
Steven A Godwin, MD, FACEP,
Virginia, Charlottesville, VA.
Assistant Professor and Residency
Assistant Professor and Emergency
Director, Combined EM/IM Program,
Peter DeBlieux, MD, LSUHSC
Medicine Residency Director,
University of Maryland, Baltimore,
Professor of Clinical Medicine; LSU
University of Florida
MD.
September 2006
Volume 8, Number 9
Authors
Melissa W Costello, MD
Assistant Professor of Emergency Medicine,
University of South Alabama, Mobile, Alabama
Alan Heins, MD
Assistant Professor of Emergency Medicine,
University of South Alabama, Mobile, Alabama
Daniel A Zirkin, MD
Resident Physician, Department of Emergency
Medicine, Emory University, Atlanta, Georgia
Peer Reviewers
Frank Lovecchio, DO, MPH, FACEP
Medical Director, Banner Good Samaritan Regional
Poison Center; Research Director, Maricopa Medical
Center, Department of EM; Associate Professor, AZ
College of Osteopathic Medicine
Robert Barish, MD
Professor of Emergency Medicine, Vice Dean for
Clinical Affairs, University of Maryland School of
Medicine, Baltimore, MD
Charles Stewart, MD, FAAEM, FACEP
Emergency Physician, Colorado Springs, CO
CME Objectives
Upon completion of this article, you should be able to:
1. Name the types of venomous snakes native to
the United States.
2. Describe the key characteristics for identifying
venomous versus non-venomous snakes.
3. Explain the grading system for envenomations.
4. Describe the indications and administration of
CroFabTM.
5. Effectively evaluate and manage severe snake
and scorpion envenomations.
6. Prevent and treat adverse effects of antivenom
administration.
7. Describe multiple resources available to assist in
the management and treatment of snake envenomations.
Date of original release: September 12, 2006.
Date of most recent review: September 1, 2006.
See “Physician CME Information” on back page.
Alfred Sacchetti, MD, FACEP,
Assistant Clinical Professor,
Department of Emergency Medicine,
Thomas Jefferson University,
Philadelphia, PA.
Beth Wicklund, MD, Regions Hospital
Emergency Medicine Residency,
EMRA Representative.
Corey M Slovis, MD, FACP, FACEP,
Professor and Chair, Department of
Emergency Medicine, Vanderbilt
University Medical Center, Nashville,
TN.
Valerio Gai, MD, Senior Editor,
Professor and Chair, Dept of EM,
University of Turin, Italy.
Jenny Walker, MD, MPH, MSW,
Assistant Professor; Division Chief,
Family Medicine, Department of
Community and Preventive Medicine,
Mount Sinai Medical Center, New
York, NY.
International Editors
Peter Cameron, MD, Chair, Emergency
Medicine, Monash University; Alfred
Hospital, Melbourne, Australia.
Amin Antoine Kazzi, MD, FAAEM,
Associate Professor and Vice Chair,
Department of Emergency Medicine,
University of California, Irvine;
American University, Beirut, Lebanon.
Ron M Walls, MD, Professor and Chair,
Hugo Peralta, MD, Chair of Emergency
Department of Emergency Medicine,
Services, Hospital Italiano, Buenos
Brigham & Women’s Hospital, Boston,
Aires, Argentina.
MA.
Research Editors
Nicholas Genes, MD, Mount Sinai
Emergency Medicine Residency.
Maarten Simons, MD, PhD,
Emergency Medicine Residency
Director, OLVG Hospital, Amsterdam,
The Netherlands.
snakebites and scorpion stings is unclear, supported
by large, but not population-based studies. The
American Association of Poison Control Centers
(AAPCC) compiles data from their Toxic Exposure
Surveillance System (TESS) and publishes a comprehensive annual analysis of all toxic exposures,
including envenomations, reported to all of the US
Poison Control Centers. These reports include information on exposures, ED use, and clinical outcomes
including death.1-2 In spite of this centralized clearinghouse/reporting system, the consensus among
experts in the field is that relying on self-reported
data in the cases of envenomations that do not
require ED visits results in a true incidence on
envenomation that is as high as three to four times
the number reported by TESS.3-5
The body of current literature on North
American snake envenomations can generally be
divided into two categories; pre-CroFabTM and postCroFabTM. The Crotalidae Polyvalent Immune Fab
Ovine (CroFabTM) was approved by the US Food and
Drug Administration in October of 2000 essentially
replacing the older Antivenom Crotalidae Polyvalent
(ACP) available since 1954. Indications and administration of CroFabTM will be discussed in detail later in
this article. Dozens of articles have been published
on the topic of snake envenomations since the
approval of CroFabTM, but the vast majority are
review articles by a small pool of authors. Very little
original research has been done since the CroFabTM
research in 1999. The research that has been done
has typically been in small populations and does not
include a single randomized, blinded clinical trial of
any medication, device, or technique for snakebite
management.
The most recent comprehensive review identified was “North American snake envenomation:
diagnosis, treatment and management” by Gold,
Barish, and Dart in the Emergency Medicine Clinics of
North America.4 This group of authors has written
extensively in the emergency medicine and wilderness medicine literature on the topic of snake envenomations. In addition to the review article above,
they participated in a four article symposium in the
Annals of Emergency Medicine6 (presenting the data
from the October 1999, North American Congress of
Clinical Toxicology’s, “Advances in the Management
of Snakebite” Symposium), and a full review of venomous snakebites in the New England Journal of
Medicine.5 Most of the recent research-based literature has focused on the limited areas of field man-
and anxious. His heart rate is 92, blood pressure 140/85, temperature 98.2°F orally, respiratory rate 22, and oxygen saturation 95% on room air. You note right eye ptosis; the pupils are
equal, round, and reactive to light, but the right eye does not
move past the midline on lateral gaze. Mild dysarthria is present. There are two tiny puncture marks on the right forearm
with 1-2cm of surrounding ecchymosis, but no swelling.
During your exam, respirations became shallower and labored,
with some snoring upper airway sounds. You realize that this
might be more than just the bite of a king snake...
S
nakes, both native and imported, and scorpions
produce clinically important envenomations in
the United States. While envenomations have long
been considered the purview of the Southeastern US
and the desert Southwest, the widespread and evergrowing population of people who maintain these
animals in collections and as “pets” has made the
presentation of a snakebite or scorpion sting possible
in any emergency department (ED) in the country.
These animals can produce devastating injuries and
must be recognized promptly and treated appropriately to prevent significant morbidity and death.
Not all envenomations need treatment. Indeed, there
are times when the risk of treatment outweighs the
benefit. However, failure to aggressively treat in the
appropriate situation may result in a disastrous outcome for the patient and for the clinician who does
not meet the standard of care expected of the emergency medicine specialist.
This article will divide the discussion of snakes
and the management of snake and scorpion envenomations into two sections. The first section will deal
exclusively with pit vipers (Crotalids), their identification, envenomation signs and symptoms, and
treatment with CroFabTM. Changes and controversies
in the care of patients with Crotalid envenomations
will also be addressed. The second section will deal
with coral snakes and the identification and treatment issues that they present, along with a brief discussion of scorpion stings and the “exotics:” The
non-native captive snakes that are found more and
more commonly in the US, both in zoos and as pets.
Critical Appraisal Of The Literature
Unfortunately, evidence from well-conducted clinical
research is in short supply to guide the management
of envenomations. The literature supporting the ED
evaluation and treatment of envenomations is generally weak, with few randomized trials or meta-analyses, and many review articles rehashing the same
weak literature. Even the epidemiology of
Emergency Medicine Practice©
2
September 2006 • EBMedice.net
agement changes, trials of venom extraction devices,
surgical management, and the expansion of the use
of CroFabTM to non-Crotalidae envenomations; all of
which will be discussed in this article.
Coral snake envenomations are not treated with
CroFabTM and thus will be reviewed separately. It is
fairly rare and human treatment studies include only
case series.7 Additionally, one well-designed animal
model study explored a technique for first aid of
coral snake envenomations.8 Similarly, the evaluation and management of scorpion envenomations is
supported by only one randomized, placebo-controlled trial from Tunisia that enrolled a group with
few severely affected people, and a systematic
review including a randomized trial and three cohort
studies.9,10
Non-native venomous snakes represent a heterogeneous group of animals kept by collectors and
zoos. These snakes cause some dangerous envenomations each year in the United States, but the
majority of the literature on evaluation and treatment of these snakes is published in the countries
where the snakes are native. Most of these reports
are low quality, but a few randomized, controlled trials support practice and are available through online
databases by searching the specific species of snake
responsible for the envenomation.
Clinical guidelines are of virtually no help in
guiding the assessment and treatment of envenomations. A search of the National Guidelines
Clearinghouse using various key words, including
snakebite, envenomation, rattle snake, coral snake,
scorpion, cobra, krait, venom, etc. yielded only one
relevant guideline, entitled, “First aid: 2005
International Consensus Conference on
Cardiopulmonary Resuscitation and Emergency
Cardiovascular Care Science with Treatment
Recommendations,” published in Circulation.11 The
two suggestions for the first aid of snakebites are:
1) First aid providers should not apply suction to
snakebite envenomation sites; this recommendation
is supported by a few Class II and III studies.
2) Properly performed pressure immobilization
is recommended for first aid treatment of Elapid
snakebites. The first aid provider creates this
pressure by applying a snug bandage that allows
a finger to slip under the bandage; this recommendation is supported by a single Class III study in
a porcine model of a coral snake envenomation.
EBMedicine.net • September 2006
Epidemiology And Etiology
Given the multitude of people who have an intense
and occasionally irrational fear of snakes, it is hard
to believe that snakebites are a relatively infrequent
occurrence. Even harder to believe, is that fatalities
from snake envenomations are exceedingly rare. It is
estimated that there are approximately 45,000
snakebites per year in the United States.12 Seven
thousand to 8000 of these bites are attributed to venomous snakes, but these bites only result in five to
six deaths annually.3,12 The Toxic Exposure
Surveillance System (TESS) report for 2004 breaks
down snakebites by species, with 98% of bites by
venomous snakes in the US from the Crotalids; there
were a total of 5046 reported bites, with 167 major
complications and two deaths. TESS reported 97
coral snake envenomations in 2004, with four major
complications and no deaths. Envenomations by poisonous exotic snakes resulted in 131 reported cases
in 2004. These bites were more severe than coral
snakebites with morbidity and mortality rates similar
to that of rattlesnakes; about 35% moderate injury,
7% major injury, and one death.
One of the major limitations of the data reported
by TESS is that, generally speaking, the data is compiled only from snake envenomations that result in
visits to a physician (ED or other) or a call to the
regional or national Poison Control Center. Many
people who sustain minor envenomations or dry
bites (bites where no venom is injected) will not seek
medical care. It is reasonable to conclude that the
patients who do not seek medical care generally will
not take the time to report their envenomation to
TESS. Thus, authors of scientific and medical writings are left to speculate on the true incidence of
envenomations. The current consensus seems to be
based on the estimate given by Parish in a 1954
report on the incidence of treated snakebites in the
US (45,000),12 and a report by Langley and Morrow
estimating that the actual incidence and death rate
from snakebites was three to four times that reported
by TESS.3 The majority of deaths occur among children and the elderly, among those for whom
antivenom is not given, is postponed, or is administered in insufficient quantities, and among members
of fundamentalist religious groups who handle poisonous snakes during religious rituals.13
Rattlesnakes, with their presence in virtually all of
the continental US and the high potency of their
venom, are responsible for the majority of snakebite
3
Emergency Medicine Practice©
fatalities with the diamondback rattlesnake accounting for 95% of these fatal bites.14-15
Snakebites are more common during the spring
and summer months when both snakes and people
are more active outdoors. Very little has changed in
the literature regarding the demographics of snake
envenomations since 1966, when Parish published
one of the first comprehensive survey results in
Public Health Reports.12 Best summarized by Gold et
al, “the majority of victims remain men between the
ages of 17 and 27 years. More than 95% of the bites
are on the extremities, and most occur between April
and October, the peak months being in July and
August.”4 Nonetheless, an emergency physician in
any part of the country can potentially see snakebites
at any time of the year, often from deliberate exposure to captive snakes.16
Of the nearly 3000 species of venomous and nonvenomous snakes worldwide, there are less than 40
species of venomous snakes in North America. To
simplify things even further for the non-herpetologist, the venomous species of North America can be
divided into four main types: Rattlesnakes (consisting of species from both the Crotalus and the
Sistrurus genus), copperheads (genus Agkistrodon),
cottonmouths (genus Agkistrodon), and coral snakes.
The first three types are members of the Viper family
and the Crotalinae subfamily; coral snakes are North
America’s only native member of the family of
snakes called Elapids.4,17 The family Elapidae also
includes many of the most dangerous snakes in the
world including cobras, kraits, adders, and mambas.
Three different species of Elapids can be found in
North America from North Carolina to Arizona and
as far south as several hundred miles south of the
US-Mexican border. The Eastern coral snake
(Micrurus fulvius) is the most dangerous of the coral
snakes. The highest concentrations are found in
Florida, but it has also been found from North
Carolina to Louisiana. The Western coral snake
(Micruroides euryxanthus) is found in the Sonoran
Desert of Arizona, northern Mexico, and the southwest corner of New Mexico below 5800 feet. The
third Elapid in the US is the Yellow-bellied sea snake
(Pelamis platurus). It has weak venom relative to the
other Elapids, but it can still be dangerous to
humans. The range of this sea snake in North
America is limited to the southernmost areas of
California and the northern pacific coast of Mexico.
Exotic snakes are held by collectors and zoos in scattered places throughout the country and may repreEmergency Medicine Practice©
sent any of the venomous species found in the
world, but are most commonly cobras, kraits, and
true vipers (as opposed to the pit vipers).
Scorpions
Scorpions are arthropods in the class Arachnida,
sharing some characteristics with spiders, and are
found worldwide. Most species of scorpions deliver
venom that is not dangerous to humans, but there
are a noteworthy few that deliver a potent, although
not deadly, venom. In the US, scorpions are found
primarily in the desert Southwest, with only one
species, the Bark scorpion Centruroides sculpturatus
(aka exilicauda), dangerous to humans. The Bark
scorpion is only found in Arizona and Northern
Mexico.
The South African and Tunisian scorpions are
very different from the Southwestern scorpion. In
the US, most envenomations are treated at home,
with very few advancing to pulmonary edema or
shock. Most of the large case series and controlled
trials have been performed outside the US, and caution must be used when extrapolating these studies
to the management of patients in the US.
Pathophysiology
Pit Vipers/Crotalids
Crotalid venom is a stunningly complex mixture of
proteins that include proteolytic enzymes, collagenases, phospholipases, nucleotidase, hyaluronidase,
acetylcholinesterase, and amino acid oxidase. In
addition, it contains elemental metals, amino acids,
carbohydrates, lipids, serotonin, and even histamine.18 Local toxic effects include swelling, pain,
ecchymosis, and blebs; systemic effects result in
coagulopathy, myocardial injury, muscular paralysis,
and central nervous system injury. 4,6 See Table 1 for
a more detailed description of venom effects.
Crotalid venom has multiple effects on many
organ systems and one of the most prominent is the
venom’s power as an anticoagulant. Recently, it has
been the subject of studies looking into clinical applications of a number of proteins isolated from venom
for development of new anticoagulant drugs. These
proteins have shown to inhibit platelet adhesion by
interfering with the binding of vWF to the GPIb
receptor and may provide an entirely new target for
antiplatelet agents.19
Venom composition will change depending on
the species of snake, age, diet, geographic location,
4
September 2006 • EBMedice.net
and time of the year.4 Several types of Crotalids, once
mature, have the ability to vary the amount of venom
injected during a strike. Generally, the younger the
snake, the more potent their venom.20
scorpion antivenom conducted in Tunisia, 82.4% of
825 patients had only local effects. The other 17.6%
exhibited systemic effects, primarily autonomic
instability, e.g. hypertension, sweating, and fever.
Nine (1%) of 825 patients experienced cardiogenic
shock, six (0.8%) had pulmonary edema, and two
(0.25%) died.23
Coral Snakes
Coral snake venom is a heterogeneous mixture of
peptides and enzymes with primarily neurotoxic
effects on nerve conduction and neuromuscular
transmission.7 The mechanism of action and pharmacodynamics of coral snake venom is unclear and
has apparently not been the subject of any published
research. It is unlikely that this uncertainty will be
addressed unless, like the Crotalids, a researcher
determines a potential clinical use for Elapid venom
or one of its components. Some cytotoxic effects
may occur but are usually minor especially when
compared to Crotalid venom. Often, there are no
symptoms for one to five hours but, once they begin,
systemic signs and symptoms may progress rapidly.
Tremors and cranial nerve dysfunction resulting in
ptosis, dysarthria, and dysphagia are common in
substantial envenomations. Respiratory depression
occurs more rarely and late in the course.
Section 1. The Pit Vipers (Family Viperidae,
Subfamily Crotalidae): Rattlesnakes, Cottonmouths (Water Moccasins), And Copperheads
Identification
Identification of the snake that produced the bite can
be extremely helpful. Accurate information about
the identity of a snake can provide information on
the potency of the venom (rattlesnake venom being
the most potent among native US snakes) and the
patient’s expected clinical course and response to
antivenom. Often, well-meaning patients or
bystanders will bring in the snake for identification;
it goes without saying that unless you have significant experience handling snakes it is best not to handle specimens that are brought in...alive or dead! If
Non-native Venomous Snakes
Imported snakes of the family Elapidae share some
of the characteristics of coral snakes, including a
tendency for elapid venom to cause
mostly neurotoxic effects. Snakes of the
family Viperidae, including Central and
South American and Asian pit vipers
and the true vipers of Africa, the
Middle East, and Europe, are similar
in effect to the Crotalid pit vipers of
North America, with primarily cytotoxic venom, resulting in local tissue damage, coagulopathy, and organ dysfunction.
Scorpions
Scorpion venom is delivered through a
tail stinger from two venom glands and
is a combination of peptides and proteins with proteolytic and neurotoxic
effects. Neurotoxic effects are proposed
to be mediated by effects at sodium and
potassium channels of neurons.21-22
Most envenomations produce only local
effects of pain and swelling. In a randomized, placebo-controlled trial of
EBMedicine.net • September 2006
Table 1: Snake Venom
Components And Their Effects
*Refers to quantity of the envenomation.
Though not comprehensive, this table is designed to provide an
easy reference in clinical practice.
Reprinted from Kiran S, Senthilnathan TA Update in Anesthesia, Issue 16 (2003) Article 6.
5
Emergency Medicine Practice©
the animal is brought in dead or in parts, keep in
mind that snakes retain the ability to strike, bite, and
potentially cause a significant envenomation for several minutes after death or decapitation. If you have
been provided a specimen that is fairly intact and is
in an appropriate container to allow for safe inspection (and you don’t develop profound cataplexy at
the thought of handling a snake), then knowing a
few good resources can be helpful in accurate identification; see Table 5 on page 18.
Figure 1 shows some of the characteristic features that help distinguish the venomous pit vipers
from the thousands of non-venomous species of
snakes in the US. In general, pit vipers will have a
triangular shaped head, elliptical pupils, and a heat
sensing pit in front of the eye. These snakes will also
have a set of retractable fangs and may or may not
have a rattle on the tail. The body markings on the
Crotalids are widely variable and can be helpful in
identifying the specific species of snake. The fourth
species of native venomous snake, the coral snake, is
in a totally different subfamily of snakes and does
not have any of this characteristic set of pit viper features despite possessing very potent venom. The
clinically important feature to know with coral
snakes is the difference in the markings between a
coral snake (venomous) and a king snake (non-venomous) which has evolved very similar markings to
the coral snake in order to fool its prey. The old saying, “red on yellow kills a fellow; yellow on black,
venom lack,” works well to differentiate the corals
from the king snakes in North America ONLY. Both
species are found throughout the geographical distribution of the coral snake.
Generally speaking, if you see more than two or
three snakebites a year or if you are the regional
referral center for snake envenomations (like we
are), you may also want to keep a field guide
specific to your region in your ED reference library.
Peterson Field Guides publishes two titles on
reptiles and amphibians, one for Eastern and
Central North America and one for the Western
US.24-25 These guides are recommended by several
herpetology sources and are inexpensive (approximately $15 each) references to keep handy. Other
potentially useful resources are poison control centers, medical toxicology divisions of large hospitals,
state agriculture departments, university veterinary
programs, and local zoos that keep and care for reptiles.
Emergency Medicine Practice©
Prehospital / Wilderness Care
Multiple methods for treating snakebites in the
field have gone in and out of vogue over the years,
and have been discussed at length both in the medical and non-medical literature. Recently, the
recommended first aid measures for snakebites
have been revised to exclude many treatments of
the past. Arterial tourniquets, aggressive wound
incision/excision, electric shock, and ice submersion/cryotherapy may all worsen a patient’s
condition.16-17 Current recommendations for field care
call for very little beyond rapid transportation; see
Table 2.
Field care by bystanders or emergency medical
services should include removing the patient from
the area where the bite occurred. It is not recommended to try to catch or kill the offending snake
simply to bring it to the hospital. In other words,
common sense and discretion should prevail. A
snake that has taken up residence under the slide on
a school playground is very different from a snake
that strikes because you stepped on its home in the
middle of the woods. A good description of the
snake in combination with the patient’s signs and
Table 2: Prehospital Care of Snake
Envenomations
6
September 2006 • EBMedice.net
Figure 1: Comparison Of Venomous Snakes (Pit Vipers)
And Nonvenomous Snakes In The US
From Gold BS, Dart RC, Barish RA. Bites of venomous snakes. N Engl J Med 2002;347(5):347-58; with permission.
EBMedicine.net • September 2006
7
Emergency Medicine Practice©
symptoms is generally adequate to initiate in-hospital treatment.
Patients should be reassured, placed at rest, kept
warm, and transported immediately to the closest
medical facility. The bitten extremity should be
immobilized and kept at or below the level of the
heart, and all constrictive clothing, jewelry and
watches should be removed.4,15,26 Closely monitor
vital signs to assess for hypotension as a sign of systemic toxicity and hypotension should prompt a
bolus of intravenous isotonic fluids.5,17 Incision and
suction, once the standard of care for the treatment
of snakebites, is no longer recommended and may, in
fact, pose more risk than benefit.27 Incision has
never been shown to be beneficial for extracting
venom and can cause unintended and potentially
disabling injury to digital nerves, arteries, and tendons even when performed by an experienced
provider.28 Oral suction of the venom is strongly discouraged both for its lack of effectiveness in removing venom and because of the possibility of introducing oral flora into the wound, potentially complicating treatment.
There are several commercial snakebite kits on
the market containing a multitude of items for the
treatment of snakebites. Generally there is some
combination of venous and/or arterial tourniquets,
lancets or scalpels, antiseptic wipes, and various suction devices. Most of the kits were developed and
marketed since before the “incision and suction”
treatment went out of vogue.
The most recognized and researched of these
suction devices is the Sawyer Venom Extractor®.
The marketing information available on the internet
for this product references its ability to remove “up
to 30%” of the snake venom based on two small
studies (one animal in 1985 and one human in 1986)
by Bronstein et al.29-30 A well-designed, 2004 human
study by Alberts et al27 using simulated radiolabeled
snake venom demonstrated minimal (0.04%) venom
extraction from a simulated snakebite wound and
only a 2% reduction of total body venom load.
Given the poor performance of this device in the
controlled trials and the elimination of most of the
former interventions from the field treatment recommendations, Johnson said it best when he stated,
“the best snakebite kit is probably the keys to a car
that runs.”15
EMS may be faced with a snakebite victim who
has had one or more well-intentioned field interventions by a bystander and it is important to know
which should be discontinued and which should
remain until hospital evaluation is complete.
Incision is one of the most likely field interventions
Table 3: Severity Grading69
Emergency Medicine Practice©
8
September 2006 • EBMedice.net
the EMS provider may encounter. In cases where an
incision has been made, control bleeding and apply a
moist dressing.17 If an extractor device is correctly in
place it should be left in place until the arrival at the
hospital.17 Arterial tourniquets should be removed
due to the potential for limb ischemia but venous
tourniquets or “constriction bands,” defined as wide,
flat bands that restrict venous and lymphatic flow to
impede absorption of venom, can be left in place.
These constriction bands, which should be loose
enough to allow two fingers to slip easily underneath, have shown some benefit in delaying
absorption of venom in experimental models and
have been suggested as therapy in patients with
prolonged transport times.17,31 If a device has been
applied and it is not causing vascular compromise, it
should be left in place by EMS during transport.
EMS personnel should frequently reassess the tension of any constrictive device to ensure that progressive limb edema does not result in a venous constriction band becoming an inadvertent arterial
tourniquet.10
omations. Patients with significant envenomations
will have considerable pain both from the local cytotoxic effects of the venom and the diffuse muscle fasciculations caused by the neurotoxins and myotoxins
in Crotalid venom. It is acceptable to give narcotics
and benzodiazepines for comfort while antivenom is
being prepared except in the cases of a coral snake,
Mojave rattlesnake, or Eastern Diamondback rattlesnake envenomation where the neurotoxic effects
of the venom can result in severely impaired mental
status. There are no good, evidence-based recommendations in the literature for choice of drugs or
dosing. NSAIDs should be avoided due to their
antiplatelet effects potentially worsening venominduced coagulopathy.
History
Once the patient arrives in the ED, key pieces of
information pertaining to the snakebite itself must be
ascertained; see Table 4. Ask the patient about
symptoms that may indicate a significant envenomation; particularly pain, numbness, nausea, tingling
around the mouth, metallic taste, muscle cramps or
fasciculations, dyspnea, diplopia, or dizziness.4,14-15,18-28
Additionally, ascertain the traditional components of
a patient history including a comprehensive review
of any co-morbid medical conditions (particularly
cardiac disease and coagulopathy), a list of current
medications, allergies (especially to papain or
papaya based extracts, latex, and horse-based or
sheep-based products), the time of last oral intake
and the patient’s tetanus status. Obtaining the history should not interfere with the initiation of treatment in the critically ill or clinically deteriorating
patient.
ED Management And Stabilization
Initial Stabilization
Upon arrival in the ED, whether by ambulance or
by personal vehicle, a rapid assessment of the
patient should obviously include an evaluation
of airway, breathing, and circulation. Barring a
situation in which these are compromised and
need to be addressed immediately, the initial stabilization of a snakebite victim includes many of the
interventions that are recommended for the field
providers.
Obtain an initial set of vital signs, place the
patient on continuous cardiac, BP, and pulse oximetry monitoring (on an unaffected extremity), remove
constrictive clothing and jewelry, establish intravenous access, and draw blood for labs including
tubes for a type and screen and coagulation studies.
Supplemental oxygen can be given on a case-by-case
basis. Use a Sharpie® or surgical marker to mark
and time the leading edge of erythema. Also, mark
two to three sites above the bite as locations for serial
measurement of limb circumference.
Once a clinically significant envenomation has
been identified, rapidly begin the process of obtaining the appropriate antivenom and begin the mixing
process, and/or arrange for rapid transfer of the
patient to a facility capable of handling snake enven-
EBMedicine.net • September 2006
Table 4: Key Questions In The History
Of A Patient With A Snakebite
• The location of the wound or wounds
• The time of the bite
• The type of snake
• Treatment provided by prehospital
bystander or EMS
• Changes in the patient’s condition since
the time of the bite
9
Emergency Medicine Practice©
tration of antivenom), liver function tests, total creatine kinase, serum myoglobin, arterial blood gases,
and chest radiography.4,14-15,18,28 This second set of tests
can be ordered judiciously on a case-by-case basis,
taking into account severity of the envenomation and
co-morbid disease. Grading severity of envenomations will be discussed in detail in the next section.
Physical Exam
Focus the initial physical exam on the evaluation of
the ABCs and provision of adequate resuscitation.
Once the adequacy of all elements of the primary
survey is established and the steps outlined in the
initial stabilization are completed, the evaluation of
the bite site can commence. As mentioned, onefourth of snakebites are dry. Examine the site of the
bite for fang marks or scratches and consider the
possibility of other types of animal bites or injuries if
the diagnosis of a snake envenomation is in doubt.
Pay particular attention to any local signs of envenomation, i.e. edema, petechiae, ecchymosis, or bullae
formation.18 Document circumferential measurements
at several sites above and below the bite site.16 Mark
a line at the site of each measurement to ensure accurate reproducibility. Repeat these measurements
every 15 to 30 minutes during the course of treatment. Also, mark and time the edge of the swelling
to serve as an index of local progression.4
Focus the remainder of the physical exam primarily on the cardiovascular, pulmonary, and neurologic systems. Patients may be hypotensive due to third
space losses and hemorrhage. Initial treatment for
hypotension is intravenous isotonic fluids.28 The
neurologic exam becomes particularly important in
severe envenomations, especially in cases of Mojave
rattlesnake, coral snake, or non-native/exotic envenomations where altered mental status and neurologic
impairment can be a significant and often delayed
feature of the envenomation.18
Compartment pressures: The other diagnostic test
that can be performed in selected cases is the measurement of compartment pressures. Most envenomations involve only subcutaneous deposition of
venom. In the rare intramuscular envenomation,
compartment syndrome may develop but it is often
impossible to distinguish the symptoms of compartment syndrome (classically the 5 “P’s:” pain out
of proportion, pallor, parasthesia, paralysis, and
pulselessness) from the symptoms of a significant
envenomation. Formerly, fasciotomy was the recommended treatment for snakebites with suspected
compartment syndrome, but current review suggests
a more conservative treatment plan involving serial
Stryker measurement for compartment pressures.
Treatment
General Principles
The treatment of patients with snake envenomations
entails aggressive supportive care and early administration of adequate doses of antivenom when indicated.4 Airway, breathing, and circulation are of primary importance. Establish peripheral intravenous
access in an unaffected extremity. Hypotensive
patients should receive fluid boluses to replace third
space losses and may require blood transfusion if a
venom-induced coagulopathy has resulted in significant hemorrhage. The transfusion of blood products
will only temporize coagulopathy and ongoing
blood losses until the venom is neutralized by
antivenom. To be clear...fresh frozen plasma does
not fix an envenomation induced coagulopathy.
Diagnostic Studies
The number and type of recommended diagnostic
studies in a patient with a snake envenomation
varies throughout the snakebite literature. However,
the majority of authors agree that a core group of
tests is indicated: A baseline complete blood count
(CBC) with platelet count, coagulation studies
including prothrombin time, partial-thromboplastin
time, activated partial-thromboplastin time, fibrinogen level, fibrin split products (fibrin degradation
products), basic electrolytes, blood urea nitrogen,
serum creatinine, and urinalysis. Patients bitten by
an unknown species with no evidence of toxicity
require, at a minimum, coagulation studies which
are necessary for the grading of the envenomation.
Various sources also recommend obtaining an electrocardiogram, a specimen for type and screen or
type and crossmatch for blood products (as crossmatch may be more difficult following the adminisEmergency Medicine Practice©
Analgesics
Consider analgesics part of the standard therapy for
envenomations. Pain control will usually require
parenteral narcotics for the first 24 to 48 hours of
therapy in patients that are receiving antivenom.33
Skeletal muscle fasciculations can be treated with
parenteral benzodiazepines.
10
September 2006 • EBMedice.net
cally break envenomations into four or five categories, ranging from “dry bite” or nonenvenomation
to severe/very severe. Table 3 on page 8 is a synthesis of several scoring systems from different authors
and can be used to grade the envenomations of all
pit viper or suspected pit viper envenomations3.
Envenomations are graded based on the symptom or
sign that places the patient in the highest (most
severe) category.4 Treat moderate and severe/very
severe envenomations (Grades II-IV) with antivenom. We have combined the Grade III and IV categories for simplicity since the treatment remains the
same and the signs and symptoms differ only in
matter of degree.
CroFab™ was FDA approved in 2000 for the
treatment of all “mild to moderate North American
Crotalidae envenomations;” however, all of the
research upon which this approval was based was
done with rattlesnake envenomations.37,40
Copperheads, while a member of the Crotalidae subfamily of vipers, are a separate species from the rattlesnakes and copperhead venom is not used in the
preparation of CroFab™. As recently as 2004,
authors have recommended against the administration of CroFab™ for copperhead bites because the
venom effects were not “serious enough” and the
risks of antivenom outweighed the benefits.40 One
retrospective review of copperhead bites by a group
at Carolinas Medical Center showed that CroFab™
administration resulted in an improvement in local
symptoms.41 Unfortunately, this was a small study
involving only 32 patients. The selection criteria for
treating with antivenom were not standardized and
the “treatment group” represented only 8% of the
copperhead envenomations that presented to
Carolinas Medical Center during the review period.
A larger, multicenter, randomized controlled study of
CroFab™ in the treatment of copperhead bites is certainly warranted and needs to include comparisons
not only of local symptoms, but also of short- and
long-term disability and adverse events associated
with treatment versus non-treatment.
After the severity of the envenomation has been
determined, patients with an indication to receive
CroFab™ should have the drug administered as soon
as possible. Early (< 6 hrs) administration of
CroFab™ has been clinically shown to reduce clinical
decline and systemic coagulopathy. Reassess and
admit these patients to the ICU for monitoring and
the completion of the CroFab™ treatment.
When the possibility of an envenomation by a
Wound Care
Even in the absence of a clinically significant envenomation, snakebites are puncture wounds and therefore require local wound cleansing and tetanus prophylaxis. Interestingly, there is good data to show
that snakebites have very low rates of infection.34-35
In spite of a broad spectrum of oral flora cultured
from the mouths of snakes, the venom is postulated
to have antibacterial properties.36 Based on these
studies, prophylactic antibiotics are not currently recommended for snakebites.33
Compartment Syndrome
In cases with documented pressures above 30mmHg,
give an additional four to six vials of antivenom
along with 1 to 2gm/kg of mannitol over 30 minutes;
limb elevation and re-evaluation of pressures after
one hour should occur prior to consideration of fasciotomy.4,28,32 It is recommended that the appropriate
service that handles fasciotomy in your facility (generally, surgery or orthopedics) be consulted early in
the course of treatment despite the rarity of envenomations that ultimately require this procedure.
Crotalidae Polyvalent Immune Fab (CroFab™)
CroFab™ was approved by the FDA in October of
2000. CroFab™ is a preparation of ovine Fab (monovalent) immunoglobulin fragments obtained from the
blood of healthy sheep flocks. These sheep are
immunized with one of the following North
American snake venoms: Crotalus atrox (Western
Diamondback rattlesnake), Crotalus adamanteus
(Eastern Diamondback rattlesnake), Crotalus scutulatus (Mojave rattlesnake), and Agkistrodon piscivorus
(Cottonmouth or Water Moccasin).37 Note that copperhead venom is not included in the immunizations.
According to the CroFab™ package insert,
patients with allergies to papain, chymopapain, other
papaya extracts, or the pineapple enzyme bromelain
may be at risk for an allergic reaction to CroFab™. In
addition, it has been noted in the literature that some
dust mite allergens and some latex allergens share
antigenic structures with papain and patients with
these allergies may be allergic to papain.38-39
Indications: Initiating treatment of an envenomation
with CroFab™ is largely based on physical findings,
their severity, and development over time.
Classically, envenomation severity has been determined based on symptoms and quantified using one
of a few accepted grading scales. These scales typiEBMedicine.net • September 2006
11
Emergency Medicine Practice©
reconstitution of CroFab™ can begin prior to patient
arrival. Nonetheless, the eagerness to begin the
process of mixing needs to be tempered by the possibility of not needing to use a treatment which has an
average cost of nearly $1000 per vial.
The recommended initial dose of CroFab™ is
four to six vials. Dosage requirements are dependent
upon the individual patient response. The use of the
recommended adult dosages in the pediatric population appears to be safe.43-44 The initial dose should
not be reduced in a pediatric patient because, despite
their smaller size, the volume of venom to be neutralized is not reduced. Anecdotal experience with
the older ACP venom indicates that antivenom
requirements may, in fact, be higher in pediatrics; but
no clear clinical correlation between age, size,
weight, or snake species and antivenom requirement
has ever been demonstrated with either antivenom
preparation. However, the fluid status of the pediatric patient should be taken into account when
deciding on the initial dilution of the dosage to be
given. Most children can handle the 250cc volumes
without difficulty as this correlates to a 20cc/kg
bolus for a 12.5kg child, but fluid status may become
an issue in children less that 10kg.44 Unfortunately,
there are no controlled studies to provide guidance
on the delivery of higher concentrations.
Deliver the initial dose at a rate of 25 to 50mL/hr
for the first ten minutes. If no acute allergic reaction
is noted after this initial time; deliver the remainder
of the first four to six vial dose at the full rate of
250mL/hr. If a reaction occurs, administer both H1and H2-receptor blockers. If the symptoms resolve
and the reaction was mild, the infusion can be continued with close monitoring. In a retrospective
review of CroFab™ safety by Dart and McNally, the
two patients with severe reactions (cough and widespread urticaria +/- wheezing) had infusions discontinued, diphenhydramine and an H2-receptor
blocker given, and then infusions restarted. In one
patient the reaction recurred and no further antivenom was given. In the other patient, the infusion was
restarted and completed without incident with the
precaution of an epinephrine infusion which is only
described as being administered “in standard
doses”45-46 or as a “low dose infusion” which was
stopped 30 minutes after the completion of the infusion of CroFab™.47
After the initial dose of four to six vials, “initial
control” is achieved when an adequate clinical
response has occurred, i.e. no further progression of
coral snake or Mojave rattlesnake exists, the scale
presented in Table 3 should not be used. The signs
and symptoms of such venom may be significantly
delayed and have their primary effects on the nervous system. In these cases, extend the observation
period to at least 12 hours from the time of the injury
to account for this delay.4-5,18 Confusing matters
somewhat, some authors recommend admission of
all suspected coral snake envenomations for close
monitoring.4 Presumably the authors are differentiating “observation” in the ED from “admission” meaning 23 hours or more. Unfortunately, there is no
good clinical data upon which these recommendations are based.
Preparation and Administration: If possible, obtain
written, informed consent prior to the administration
of CroFab™, advising the patient primarily of the
risks of anaphylaxis, delayed serum sickness, and/or
death. In the initial clinical studies of CroFab™, the
rates of allergic reaction were low and skin testing
was not done. Skin testing done in the setting of the
older polyvalent antivenom administration (which
had a much higher anaphylaxis rate) has been shown
to have both high false-positive rates and false-negative rates, 33% and 10 to 36% respectively. As clinical experience and comfort with CroFabTM have
increased, the need for skin testing prior to administration has never been shown to be useful.
Additionally, skin testing delays the onset of definitive treatment and the risk of true anaphylaxis with
CroFab™ is substantially lower than with the older
Antivenom Crotalid Polyvalent (ACP).42 Despite this
data, the treating physician should always be prepared for an anaphylactic reaction to antivenom with
basic supplies such as epinephrine, diphenhydramine, airway equipment, oxygen, and pressors.
CroFab™ is packaged as a shelf-stable
lyophilized powder in a vial. Reconstitute each vial
of CroFabTM with 10mL of Sterile Water for Injection
USP (diluent is not included). After being thoroughly mixed, further dilute each of the reconstituted
vials to be used in a given dose in a single 250mL
bag of 0.9% Sodium Chloride USP. Use this reconstituted and diluted product within four hours of mixing. The reconstitution and dilution process is time
consuming; even in the most practiced hands, preparation of an initial dose of CroFab™ takes more than
30 minutes. Given this, our recommendation is that
the mixing of CroFab™ should begin as soon as the
patient demonstrates evidence of a significant envenomation. Occasionally, in select scenerios, when
patients are being transported for treament, the
Emergency Medicine Practice©
12
September 2006 • EBMedice.net
any local symptoms, systemic symptoms, or coagulopathy. Observe the patient while administering
two additional vials every six hours for three additional doses. If, after completion of the initial dose,
the patient’s symptoms continue to progress (worsening of the local injury or systemic effects such as
muscle fasciculation, parasthesias, abnormal mental
status, tachypnea, tachycardia, or hypotension), an
additional four to six vials is recommended. This
additional dose is also warranted if laboratory studies show prolonged coagulation times, decreasing
fibrinogen levels, or worsening platelet count. If a
second four to six vial dose is given, the patient
needs to be reassessed for “initial control” and a
third round of four to six vials may be needed in the
most severe cases. Multiple studies of CroFab™
indicate that the dose required for initial control can
range from 4 to 18 vials. If three rounds of four to
six vials fail to achieve control of an envenomation,
consider an alternative diagnosis, such as another
type of envenomation, a toxic overdose, or an exotic
species.
The elimination half-life for CroFab™ is estimated to range from 12 to 23 hours.37 Further study has
shown that Fab molecules have a shorter half-life
than the IgG used in traditional polyvalent antivenom.45 Early studies of CroFab™ reported cases of
recurrence of signs and symptoms of the original
envenomation after intital control. This may be due
to a Fab half-life that is shorter than the elimination
half-life of snake venom, thus the administration of
CroFab™ includes three additional doses to prevent
symptom recurrence. Once initial control is
obtained, admit the patient to the ICU and give two
vials of CroFab™ at 6, 12, and 18 hours after the
completion of the control doses.
incidence and severity of reactions to CroFab™
appears to be significantly reduced. In the initial
clinical studies discussed above, seven of the first 42
patients treated had an early, though relatively
minor, reaction (five urticaria, one cough, one
urticaria/dyspnea and wheezing). However, it is
important to note that there have been case reports
of serious reactions and serum sickness related to
CroFab™.46-47
Disposition
Patients who have no findings of envenomation (dry
bites/Grade 0) can be discharged after a four to six
hour observation period.18 Patients who have limited
local signs of envenomations but do not meet the criteria for CroFab™ need to be monitored for progression of symptoms in the ED or observation unit for
a minimum of 8 to 12 hours from the time of the
bite4-5,18,28 Although most patients with Crotalidae
envenomations begin to manifest local symptoms
within 10 minutes of a bite and virtually all have
some findings within 30 to 60 minutes, there are some
patients who will not manifest symptoms for several
hours, although the mechanism for this is unclear.4 In
those cases where the patient does not initially meet
treatment criteria, observation and intermittent
reassessment is critical since local findings or symptoms that would upgrade the envenomation into a
treatment category may be absent initially.
Patients who receive CroFab™ are admitted to
the hospital as discussed in the previous section. The
half-life of CroFab™ is less than that of polyvalent
antivenom. This fact, coupled with the persistent
activity of the depot-style delivery of the venom, can
lead to recurrent coagulopathy despite appropriate
antivenom treatment.68 Such recurrence is characterized by decreased platelets, elevated prothrombin
time, and decreased fibrinogen. During the initial
trials, such a recurrent coagulopathy was observed
only in those patients experiencing coagulation
abnormalities at initial presentation. After initial
treatment, this coagulopathy may be present for
weeks. Therefore, post treatment, monitor those
patients presenting with coagulopathy for signs and
symptoms of recurrent coagulopathy after discharge
every two to three days until the coagulopathy
resolves48-49 Advise patients to contact their physician
immediately if they experience new symptoms or
unusual bruising or bleeding after hospital discharge
as additional antivenom treatment may be needed.
Monitoring: After stabilization and initial control of
the patient’s symptoms have been achieved, and the
administration of the scheduled doses has begun,
continue close surveillance of the patient’s condition.
Observation includes monitoring of limb circumference, both above and below the bite, and using a pen
to outline the edematous area every 30 to 60 minutes.
Obtain laboratory determinations of the patient’s
coagulation status every four hours.28 If symptoms
or laboratory data warrants, the treating physician
can give an additional two vials. Any unscheduled
re-dosing after initial control does not reset the 6, 12,
18 hour schedule to the beginning.
When compared to polyvalent antivenom, the
EBMedicine.net • September 2006
13
Emergency Medicine Practice©
Emergency Medicine Practice©
14
September 2006 • EBMedice.net
EBMedicine.net • September 2006
15
Emergency Medicine Practice©
Emergency Medicine Practice©
16
September 2006 • EBMedice.net
Boyer et al recommend retreatment with two vials
of CroFab™ in the event of fibrinogen level
< 50mcg/mL, platelet count < 25,000, INR > 3.0,
aPTT > 50 seconds, multicomponent coagulopathy,
worsening trend in patients with prior severe coagulopathy, high risk behavior for trauma, or comorbidities that increase the risk of hemorrhage.49
Instruct patients to report any signs or symptoms of delayed allergic reactions or serum sickness
after hospital discharge. Serum sickness is a delayed
hypersensitivity reaction to antivenom characterized
by fever, rash, arthralgias, and lymphadenopathy
which typically begin 7 to 21 days following antivenom administration. It is much more rare with
CroFab™ than with the older polyvalent antivenom
(3% versus more than 80%) and responds well to a
tapering course of prednisone starting at 60mg daily
and tapering over seven to ten days.4,33,45 Serum sickness is the only indication from steroids in the treatment of snake envenomation.
toms of compartment syndrome and it is clinically
impossible to establish the difference without direct
measurement of compartment pressures.53 Even in
the cases where compartment pressures exceed 30 to
40mm Hg, additional antivenom administration
(four to six vials) has been shown to reduce compartment pressures, avoiding an unnecessary and disfiguring fasciotomy. Fasciotomy should only be considered in the extraordinarily rare cases where additional antivenom doses have failed to reduce measured
pressures.
In the authors’ opinion, given the ”assess, treat,
reassess, treat” management strategy, and the scheduled dosing nature of current management, a medical
ICU service may be more appropriate than a surgical
service. Clearly, in the rare case of true elevated compartment pressures, a consultation to orthopedics or
surgery is warranted. That said, snake envenomations present such a complex interaction of multisystem toxicologic effects that the determination of the
best admission service should be handled on a caseby-case and hospital-by-hospital basis.
Controversies
Alternative Treatment
Several studies have addressed specific techniques
used in the care of snake envenomations and have
shown that many of the devices and techniques traditionally used in care of snake envenomations were
either of no benefit or were, in fact, harmful.
Incisions, suction devices, packing in ice (cryotherapy), application of heat, and even application of electrical shocks have all historically been used to treat
snake envenomations. While the application of a
stun-gun or “gasoline engine spark plugs” to a
patient may be intriguing, there is no clinical benefit,
and a handful of case studies show the danger of this
therapy, even in controlled settings.50 As discussed
in detail above, little role remains for any field treatment beyond immobilization, reassurance, and rapid
transport.
Section 2. Coral Snakes (Elapidae), Exotic
Snakes, And Scorpions
Prehospital Care
As with Crotalid envenomations, there are no specific prehospital interventions for scorpion or nonCrotalid snake envenomations, other than removal
from danger, and rapid, safe transport. Immobilization of the affected limb may be helpful.
An interesting experiment testing a novel immobilization technique in a porcine model of a coral
snake envenomation has been published.8 The pressure-immobilization technique is not a tourniquet,
but uses an elastic bandage applied from the envenomation site and extended proximally. The goal is to
impede lymphatic flow, not venous or arterial flow.
Apply the bandage about as tight as a wrap for an
acute sprain, yet loose enough to allow a finger to be
inserted between skin and bandage without difficulty. Also, splint the limb to limit motion. Elapid
snakebites in other countries, primarily Australia, are
treated with a similar pressure immobilization technique.54-55 Because of the low quality evidence, this
technique must be considered experimental, even
though it has been included in a clinical guideline
for the snakebite section of an emergency care guide-
Fasciotomy
Once considered part of the primary treatment of
snake envenomations, fasciotomy is the other controversial issue that has yet to be addressed in this discussion. Good clinical data from animal studies
have shown that in snake envenomations, rabbits
that receive fasciotomy (with or without antivenom)
have poorer outcomes than those treated with
antivenom alone.51-52 The signs and symptoms of a
significant envenomation closely mimic the symp-
EBMedicine.net • September 2006
17
Emergency Medicine Practice©
line, the 2005 International Consensus Conference on
Cardiopulmonary Resuscitation and Emergency
Cardiovascular Care Science with Treatment
Recommendations.11
occur over a few minutes. Tremors, ptosis, dysphonia, dysphagia, and decreased deep tendon reflexes
are the most common neurologic signs and may indicate impending respiratory paralysis.7
Emergency Department Evaluation
Non-native Venomous Snakes
Depending on the species, envenomations may produce mostly neurotoxic effects, especially from the
other Elapids, e.g. cobras, kraits, and mambas.
Cytotoxic and coagulopathic effects may predominate from bites of other snakes, especially the
Viperidae (both pit and true vipers). Once identification is made, a poison center consultation and/or
accessing one of the available web resources will
help guide clinical decision making; see Table 5.
Coral Snakes
Coral snakebites produce little, if any, pain and local
reaction. Assessment of signs of neuropathy must be
aggressively sought, as progression to bulbar paralysis and respiratory failure may be rapid. The onset
of neurological symptoms may be delayed one to
four hours, but once neurological signs appear, progression to paralysis and respiratory failure may
Table 5: Useful Websites For Snake
Identification And Bite Management
Black and white photographs of snakes do very little to help in the identification of snakes and thus, are not reproduced in this article.
Fortunately, the internet has proven to be extremely useful for snake identification with many sites that provide a variety of full color
pictures and tools to help identify venomous snakes and differentiate them from their non-venomous counterparts. Using a search
engine and the topic "snake" or "snake identification" will result in websites that discuss the snakes that are indigenous to your region
and will aid in identification. This table lists some of the useful websites for snake identification and bite management.
• http://mdg.ext.msstate.edu/Tom_Snake/
index.html: One of the most user friendly sites
as it has been designed for identification of
snakes by the non-herpetologist. Snakes are
identified by name or features. It is based at
Mississippi State University and will identify the
snakes of the Southeast and the majority of the
snakes nationwide.
unknown snake, but if you have a good idea
of what you are looking for, this is an excellent
site for quick confirmation.
• http://trailquest.net/SNpoi.html: Avid hikers will
recognize this site as an excellent resource for all
things related to hiking. It also happens to contain a complete list of "poisonous" and "nonpoisonous" snakes that may be encountered while hiking throughout the continental United States.
Like the previous site, it requires that you choose
a species to see a photo, but it does have better
descriptions of habitat and distributions of species
by state/region.
• www.snakesandfrogs.com: This site provides
excellent descriptions (with photos) of the
various characteristics that identify snakes.
Follow the links to "How to Identify Snakes."
It is a South Carolina based site that will work
for most snakes indigenous to the US East and
Southeast. It is an excellent site if you have
the specimen in front of you but tougher to navigate if you are working solely from a patient’s
description.
• www.aza.org: The American Zoo and Aquarium
Association (AZA) can be particularly helpful
when exotics are involved in an envenomation.
The AZA maintains the Antivenom Index, a list of
the types and locations of all species-specific
antivenoms for any of the venomous exotics
housed in US zoos and larger private collections.
The Antivenom Index is a members-only list, but
national and regional Poison Control Centers
maintain active memberships and can access the
information needed to provide patient care
(American Association of Poison Control Centers,
www.aapcc.org, 800-222-1222).
• http://www.pitt.edu/~mcs2/herp/SoNA.html:
This site’s common name is "Snakes of North
America" and it can be found easily by searching
"snake identification" on Google. It is a comprehensive list of every snake native to North
America with a picture. Each snake only has
one photo and requires that you choose the
species in order to see the photo. This can
prove to be a challenging way to identify an
Emergency Medicine Practice©
18
September 2006 • EBMedice.net
Non-native Venomous Snakes
Again, guidance on testing will depend on the identification of the snake. Bites from other members of
the family Elapidae will likely require no testing.
Patients with bites from other Viperidae snakes will
probably require testing similar to that recommended previously for Crotalid envenomations, e.g. complete blood count, comprehensive metabolic panel,
prothrombin time, partial thromboplastin time, fibrinogen level, blood type and antibody screening.
Compartment pressure measurement is indicated if
compartment syndrome is suspected.
Scorpions
Children are at the greatest risk of severe effects
because of low body weight to venom ratios. Elderly
people may also have lower reserves against the
physiologic insult of the toxins. Evaluation of local,
systemic, and neurological effects is essential.
Deaths occur from cardiopulmonary collapse or respiratory paralysis. Identification of impaired tissue
perfusion, tachycardia, tachypnea, hypoxemia,
hypotension, agitation, altered mental status, and/or
cranial and somatic neuromuscular dysfunction are a
prompt for aggressive treatment. A retrospective
study of 428 patients admitted to the intensive care
unit in Tunisia for scorpion envenomations found
that respiratory rate > 30 breaths per minute, agitation, and sweating were predictors of pulmonary
edema.56
Scorpions
A complete blood count and plasma protein concentration may help predict the presence of pulmonary
edema, as will a chest x-ray. Cardiac enzymes may
demonstrate cardiac damage. Liver enzymes may
show liver damage which was correlated with poor
prognosis in a study of 951 patients admitted to a
Tunisian ICU.57 Echocardiography may be necessary
to evaluate cardiogenic shock or suspected cardiac
dysfunction.
Diagnostic Studies
Coral Snakes
There is no adequate evidence to support the use of
any specific laboratory or imaging tests in cases of
coral snake envenomations. Since there are generally
few cytotoxic effects, there is probably no need for
testing. However, if the patient requires endotracheal intubation and mechanical ventilation for respiratory failure, a chest x-ray to assess tube position
and arterial or venous blood gases to assess ventilation status are indicated.
Treatment
Prompt administration of specific antivenom, when
available and indicated, and supportive care of airway, breathing, circulation, and neurologic function
are the foundation of ED care for these envonomations and stings. Tetanus prophylaxis should be
Key Points
1. Venomous snakes (domestic and imported) and
scorpions can produce devastating injuries and
must be recognized promptly and treated appropriately to prevent morbidity and death.
5. Zoos maintain a stock of antivenoms for many
exotic, venomous snakes and may be a source for
treating envenomations from those snakes.
6. Your local poison control center has access to multiple sources for identifying venomous snakes
and obtaining specific antivenom. Call them.
2. Identification of the snake that inflicted the bite is
important but not essential for appropriate management
7. Children and elderly are at the highest risk of
severe morbidity and death from scorpion stings
and require close observation.
3. Crotalid envenomations are graded based on the
most severe sign or symptom, and CroFab™
should be administered for all moderate and
severe envenomations.
8. Equipment and medications for management of
anaphylaxis should be in place during the administration of any antivenom.
4. Once a coral snake bite is confirmed, antivenom
should be given immediately, even if no symptoms are present.
EBMedicine.net • September 2006
19
Emergency Medicine Practice©
offered if there is no clear history of tetanus immunization within five years and encouraged if it has
been more than ten years. Critical care services
including mechanical ventilation, pressor and
inotrope administration, intravenous hydration
and nutrition, and prolonged sedation are sometimes
required in severe envenomations. No credible evidence supports the use of prophylactic antibiotics or
steroids in scorpion, coral snake, or non-native venomous snake envenomations. Consideration of special needs for individual cases is discussed later in
this article.
Adverse effects of antivenom administration,
often anaphylaxis, occur in the majority of patients
who receive antivenom derived from animal serum.
Universal preparation to treat these reactions is
required for all patients receiving antivenom. Two
prospective case series, one from Australia and one
from South Africa, found over a 70% occurrence of
immediate hypersensitivity reactions, with about
half of these anaphylaxis, after administration of
(non-Fab derived) snake antivenom.58-59 In a study of
181 patients who received Antivenom Crotalid
Emergency Medicine Practice©
Polyvalent (ACP), 56% experienced a rash 3 to 21
days after antivenom with several experiencing subjective fever, itching, and arthralgias as well. These
findings appeared dose related with nearly all
patients receiving 30 or more vials of antivenom
experiencing a rash. Serum sickness, discussed previously, is much more common with the horse serum
derived antivenoms used for coral snake, exotic
snake, and scorpion envenomations than with
CroFab™.4 In a prospective observation study of
116 patients receiving scorpion (Centruroides)
antivenom for severe envenomations, four patients
had immediate reactions: Three cases of rash and one
case of anaphylaxis. Follow-up of the 99 patients was
conducted at one year; 61% experienced delayed
hypersensitivity reaction and serum sickness, which
responded to steroids and antihistamines.60
A prednisone taper, beginning at 60mg per day
over seven to ten days, along with oral antihistamines, is the most common treatments for serum
sickness.61 One randomized, controlled trial and a
systematic review of that trial concluded that the
administration of 0.25mg of 1:1000 epinephrine sub-
20
September 2006 • EBMedice.net
cutaneously in the forearm immediately before
antivenom infusion is started markedly reduced the
incidence of immediate hypersensitivity reactions;
absolute risk reduction 30%, number-needed-to-treat
was 3.3, with no significant adverse effects attributable to epinephrine.62-63
before and after depletion of stocks of antivenom,
suggest that use of antivenom in children may prevent some hospital and intensive-care unit admissions.64-65
Coral Snakes
Place coral snake specific antivenom at the bedside
of all patients with a suspected coral snake bite.
Observe and treat at the first sign of envenomation,
however minor. The antivenom is derived from
horse serum and may result in an immediate or
delayed hypersensitivity reaction. The incidence data
for adverse reactions is not available.
A provocative pilot study was conducted in children
admitted to the hospital for a scorpion envenomation, using a before-after, quasi-experimental analysis
of an intervention using prazosin, an alpha-receptor
blocker. Unfortunately, this study was done in
Tunisia, involving severe envenomations by North
African scorpions and a control therapy using
insulin and dextrose which is not standard in the US.
Prazosin dosing was 30 micrograms/kg/dose orally
at the time of presentation and repeated three hours
after the first dose and every six hours thereafter as
needed to alleviate signs of autonomic instability.
Before beginning the prazosin intervention “standard therapy” was to use an insulin and dextrose
mixture, intravenous fluids, and treatment of associated complications. Standard therapy was given in
addition to prazosin. With 20 patients in the “before
cohort” and 16 patients in the “after cohort,” the
authors reported that the prazosin group experienced fewer episodes of hypoglycemia and hyperkalemia; although, it is unclear if this is an effect of
the scorpion venom or a side effect of the
insulin/dextrose therapy. Only one of 16 patients in
the prazosin group died, compared to seven of 20 in
the control group. Hospital length-of-stay was
reduced in the prazosin group from a mean of 71.5
hours to 46.3 hours.66 With the unavailability of scorpion antivenom in Arizona, there is a pressing need
for a well designed, randomized, controlled trial of
prazosin in the treatment of scorpion envenomations
with systemic signs.
A randomized clinical trial of scorpion antivenom is currently being conducted. A two-year clinical
trial of 50 patients, supported by a FDA grant to the
University of Arizona, to study the effect of a new
scorpion FAB fragment antivenom has been completed but was only reported in a Tuscon, Arizona newspaper.67 The drug, Anascorp™, is made in Mexico
and not yet approved for use by the FDA. The clinical trial overseen by researchers at the University of
Arizona and the Arizona Poison Center in Tucson
reported that the drug used in > 100 Arizona children demonstrated benefit.
Controversies/Cutting Edge
Non-native Venomous Snakes
Early and sufficient antivenom administration is the
key to treatment of non-native venomous snakebites.
Again, positive identification of the snake is essential
so that specific antivenom can be obtained. Local
zoos are required to store antivenom, when available, for every venomous species in their collection.
Call your local zoo for availability of antivenom or
call the American Zoo and Aquarium Association
(301-562-0777) for access to their antivenom index.
Collectors of venomous snakes are not bound by
these same regulations, so a search for antivenom
must be conducted, after identification of the snake,
through local poison control centers or the American
Association of Poison Control Centers (800-222-1222).
Scorpions
Supportive care is the cornerstone of treatment.
Scorpion antivenom for Centruroides sculpturatus (aka
exilicauda), the only scorpion species in the US dangerous to humans, was previously available only in
Arizona, but production stopped in 2001. Stocks
became outdated in 2004, and are not FDA approved
for the treatment of scorpion envenomations. In
addition, there are contradictory findings from
studies of the effectiveness of antivenom in treating
scorpion stings. Complicating this, is the fact that
most studies have been done on non-North
American scorpion envenomations. For example,
one randomized, placebo-controlled trial in Tunisia
and a systematic review including the randomized
trial and three cohort studies concluded that there
was no benefit to administering antivenom.9-10 More
relevant to US practice, but weaker evidence, a case
series and a natural before-after analysis of cohorts
EBMedicine.net • September 2006
21
Emergency Medicine Practice©
Case #1: This case represents a very severe envenomation (Grade III/IV). This patient needed 12 vials of
CroFab™ to achieve initial control and resolve the coagulopathy. He was admitted to the medical ICU and monitored for several days. His pain was controlled with narcotics and benzodiazepines; he ultimately did well and was
discharged home. He did not get a new pair of boots.
Case #2: This patient required eight hours of observation. The bite was a “Grade 0” at presentation and he
remained asymptomatic for the eight hour stay. The
patient was discharged home after wound cleansing and
administration of tetanus prophylaxis.
Case 3: In this case, the patient was experiencing
neurotoxic effects of a coral snake envenomation and was
at a high risk for respiratory failure. Three vials of North
American coral snake antivenom were given, with additional doses made available at the bedside. Aggressive
supportive care, including endotracheal intubation and
mechanical ventilation were required. The patient made an
uneventful recovery. He decided to hire someone the next
time the brush needed clearing.
Disposition
Coral Snake
Any time a patient with a coral snake envenomation
exhibits severe systemic or neurological signs, admit
the patient to the intensive care unit. If the patient
does not exhibit any systemic symptoms or signs at
the end of a 12 hour ED observation period, the
patient may be discharged home.4
If the patient has systemic signs after four hours
and is requiring any supportive care, admit the
patient for at least a 24 hour observation period. At
the end of the observation period, patients no longer
requiring supportive care may be discharged.
Patients with continuing systemic signs and supportive needs should continue to receive antivenom and
be managed in an ICU setting.
Non-native Venomous Snakes
Because of the wide variety of snakes and venom
effects, no firm disposition criteria can be determined. Consult a poison control center to assist in
medical management decisions.68
References
Scorpions
Any time a patient with a scorpion envenomation
exhibits severe systemic signs, admit the patient to
the intensive care unit. If the patient does not exhibit
any systemic symptoms or signs at the end of a four
hour ED observation period, the patient may be discharged home with no specific discharge instructions. If the patient has systemic signs or requires
supportive care, admit for at least a 12 to 24 hour
observation period. At the end of the observation
period, patients no longer requiring supportive care
may be discharged.
Evidence-based medicine requires a critical appraisal
of the literature based upon study methodology and
number of subjects. Not all references are equally
robust. The findings of a large, prospective, randomized, and blinded trial should carry more weight
than a case report.
To help the reader judge the strength of each reference, pertinent information about the study, such
as the type of study and the number of patients in
the study, will be included in bold type following the
reference, where available.
1.
Conclusion
There are approximately 8000 venomous snakebites
in the US each year. The vast majoity of these bites
are from rattlesnakes, copperheads and water
mocosins. Management is based on early recognition, envenomation assessment, and administration
of the appropriate antivenom. Current concepts in
care have relenquished incision and suction techniques to history books and have promoted antivenom that is immunotherapy based.
The three cases presented at the beginning of this
article illustrate scenarios that may confront any
emergency medicine physician. The case outcomes
show the benefit of proper clinical management.
Emergency Medicine Practice©
2.
3.
4.
5.
22
Watson WA, Litovitz TL, Klein-Schwartz W, et al. 2003
Annual Report of the American Association of Poison
Control Centers Toxic Exposure Surveillance System.
Am J Emerg Med 2004;22(5):335-404. (Prospective;
2,395,582 patients)
Watson WA, Litovitz TL, Rodgers GC, et al. 2004
Annual Report of the American Association of Poison
Control Centers Toxic Exposure Surveillance System.
Am J Emerg Med 2005;23(5):589-666. (Prospective;
2,438,644 patients)
Langley RL, Morrow WE. Deaths resulting from animal attacks in the United States. Wilderness Environ
Med 1997;8:8-16. (Expert opinion)
Gold BS, Barish RA, Dart RC. North American Snake
Envenomation: diagnosis, treatment and management.
Emerg Med Clin North Am 2004;22(2):423-43. (Review)
Gold BS, Dart RC, Barish RA. Current Concepts: Bites
September 2006 • EBMedice.net
6.
7.
8.
9.
10.
11.
12.
13.
14.
15.
16.
17.
18.
19.
20.
21.
of Venomous Snakes. N Engl J Med 2002;347: 347-356.
(Review)
Dart RC, Waeckerle JF. Introduction: “Advances in the
Management of Snakebite” Symposium. Ann Emerg
Med 2001;37:166-167. (Expert opinion)
Kitchens CS, Van Mierop LH. Envenomation by the
Eastern Coral Snake (Micrurus fulvius fulvius). A
study of 39 victims. JAMA. 1987;258(12):1615-1618
German BT, Hack JB, Brewer K, et al. Pressure-immobilization bandages may delay toxicity in a porcine
model of eastern coral snake (Micrurus fulvius fulvius)
envenomation. Ann Emerg Med 2005;45(6):603-608.
(Animal study, randomized, controlled)
Abroug F, El Atrous S, Nouira S, et al. Serotherapy in
scorpion envenomation: a randomized controlled trial.
The Lancet 1999;354:906-909. (Prospective, randomized, controlled; 825 patients)
Foex B. BestBETs: Scorpion envenomation: Does
administration of antivenom alter outcome? Accessed
25 May 2006 at http://www.bestbets.org/cgibin/bets.pl?record=00567 . (Systematic review)
ECC Committee, Subcommittees and Task Forces of
the American Heart Association. First aid: 2005
International Consensus Conference on
Cardiopulmonary Resuscitation and Emergency
Cardiovascular Care Science with Treatment
Recommendations. Circulation 2005;112(22 Suppl):1525. (Consensus conference report)
Parrish HM. Incidence of treated snakebites in the
United States. Public Health Rep 1966;81:269-276.
(Survey)
Russell FE, Banner W. Snake venom poisoning. Conn’s
Current Therapy. Rakel RE (ed). Philadelphia, WB
Saunders Co, 1988, p 1002.
Juckett G, Hancox JG. Venomous Snakebites in the
United States: Management Review and Update. Am
Fam Phys 2002;65(7):1367-1374. (Review)
Johnson CA. Management of Snakebite. Am Fam
Phys 1991;44(1):174-180. (Review)
Liebelt EL, Kazzi ZN. Timing and Toxicology Critical
for Treating Snakebite Victims. UAB Insight 2006;
Spring: 9-10. (Expert Opinion)
McKinney PE. Out-of-hospital and Interhospital
Management of Crotaline Snakebite. Ann Emerg Med
2001;37(2)168-174 (Review)
Otten EJ. Venomous Animal Injuries. In: Marx JA, editor-in-chief. Rosen’s Emergency Medicine: Concepts
and Clinical Practice, 6th ed. Philadelphia: Mosby
Elsevier; 2006. 894-913 (Textbook Chapter)
Messmore HL, Jeske WP, Wehrmacher W, el al.
Antiplatelet Agents: Current drugs and future trends.
Hematol Oncol Clin N Am 2005;19:87-117. (Review)
Suncoast Herpetological Society. Venom1 Antivenom
Bank. May 2006. Accessed 12 September 2006 at
http://www.kingsnake.com/suncoastherpsociety/SH
Sven.htm.
Cheng D. Scorpion Sting. Accessed 25 May 2006 at
http://www.emedicine.com. (Review)
EBMedicine.net • September 2006
22. Rodriguez de la Vega RC, Possani LD. Current views
on scorpion toxins specific for K+ channels. Toxicon
2004;43(8):865-875. (Basic science)
23. Zuo XP, Ji YH. Molecular mechanism of scorpion neurotoxins acting on sodium channels: insight into their
diverse selectivity. Mol Neurobiol 2004;30(3):265-278.
(Basic science)
24. Conant R, Collins JT. Reptiles and Amphibians:
Eastern and Central North America 3rd ed. New
York: Houghton Mifflin; 1998. Peterson Field Guide
Series. (Reference book)
25. Stebbins RC. A Field Guide to Western Reptiles and
Amphibians 3rd ed. New York: Houghton Mifflin;
2003. Peterson Field Guide Series. (Reference book)
26. Venomous Snakes of North America. Accessed 12
September 2006 at http://www.geocities.com/
RainForest/2221/snakeid.html.
27. Alberts M, Shalit M, LoGalbo F. Suction for Venomous
Snakebite: A Study of “Mock Venom” Extraction in a
Human Model. Ann Emerg Med 2004; 43(2) 181-186.
(Prospective;8 human volunteers)
28. Dart RC, Daly FS. Reptile Bites. In: Tintinalli J, editor.
Emergency Medicine: A Comprehensive Study Guide,
6th ed. Chicago: McGraw Hill; 2004. p1200-1205.
(Textbook Chapter)
29. Bronstein AC, Russell FE, Sullivan JB. Negative pressure suction in the field treatment of rattlesnake bite.
Vet Hum Toxicol 1985;28:297 (Abstract)
30. Bronstein AC, Russell FE, Sullivan JB. Negative pressure suction in the field treatment of rattlesnake bite
victims. Vet Hum Toxicol 1986; (Abstract)
31. Burgess JL, Dart RC, Egen NB, et al. Effects of constriction bands on rattlesnake venom absorption: a
pharmacokinetic study. Ann Emerg Med 1992;21:10861093.
32. Hall EL. Role of surgical intervention in the management of crotaline snake envenomation. Ann Emerg
Med 2001;37(2):175-180. (Review)
33. Singletary EM, Rochman AS, Bodmer JC, et al.
Envenomations. Med Clin N Am 2005; 1195-1224.
(Systematic review)
34. Clark R.F, Selden BS, Furbee B, The incidence of
wound infection following crotalid envenomation. J
Emerg Med 1993;11:583-586. (Prospective, observational; 54 patients)
35. Weed HG, Nonvenomous snakebite in Massachusetts:
prophylactic antibiotics are unnecessary. Ann Emerg
Med 1993; 22:220-224. (Prospective observational; 72
patients)
36. Talan DA, Citron D.M, Overturf G.D, et
al. Antibacterial activity of crotalid venoms against
oral snake flora and other clinical bacteria. J Infect Dis
1991;164:195-198. (Basic Science)
37. Protherics. “CROFAB® CROTALIDAE POLYVALENT
IMMUNE FAB (OVINE)” Package Insert. 2006.
(Package Insert)
38. Quarre JP, Lecomte J, Lauwers D, et al. Allergy to latex
and papain. J Allergy Clin Immunol 1995; 95(4):922.
23
Emergency Medicine Practice©
53. American College of Surgery, Commission on Trauma
and American College of Emergency Physicians.
Guidelines for treatment (poster publication). Chicago,
IL: American College of Surgery; 1994. (Clinical
Consensus Statement)
54. Sutherland SK, Coulter AR, Harris RD. Rationalisation
of first aid measures for elapid snakebite. Lancet
1979;1(8109):183-185. (Expert opinion)
55. Howarth DM, Southee AE, Whyte IM. Lymphatic flow
rates and first aid in simulated peripheral snake or spider envenomation. Med J Aust 1994;161:695-700.
(Animal study, randomized, controlled)
56. Bouaziz M, Bahloul M, Hergafi L, et al. Factors associated with pulmonary edema in severe scorpion sting
patients—a multivariate analysis of 428 cases. Clin
Toxicol (Phila) 2006;44(3):293-300. (Retrospective,
cohort;428 patients)
57. Bahloul M, Chaari A, Khlaf-Bouaziz N, et al.
Gastrointestinal manifestations in severe scorpion
envenomation. Gastroenterol Clin Biol 2005;29(10):10011005. (Retrospective chart review; 951 patients)
58. Moran NF, Newman WJ, Theakston RD, et al. High
incidence of early anaphylactoid reaction to SAIMR
polyvalent snake antivenom. Trans R Soc Trop Med
Hyg. 1998;92(1):69-70. (Prospective case series; 17
patients)
59. Isbister GK, Tankel AS, White J, et al. Med J Aus.
2006;184(8):419-20. (Prospective case series; 14
patients)
60. LoVecchio F, Welch S, Klemens J, et al. Incidence of
immediate and delayed hypersensitivity to
Centruroides antivenom. Ann Emerg Med
1999;34(5):615-619. (Prospective, cohort;116 patients)
61. Lo Vecchio F, Klemens J, Roundy EB, et al. Serum sickness following administration of Antivenin
(Crotalidae) Polyvalent in 181 cases of presumed rattlesnake envenomation. Wilderness Environ Med
2003;14(4)220-221. (Retrospective;181 patients)
62. Premawardhena AP, deSilva CE, Fonseka MMD, et al.
Low dose subcutaneous adrenaline to prevent acute
adverse reactions to antivenom serum in people bitten
by snakes: randomized, placebo controlled trial. BMJ
1999;318:1041-1043. (Prospective, randomized, controlled;105 patients)
63. Nuchpraryoon I, Garner P. Interventions for preventing reactions to snake antivenom. Cochrane Database
Syst Rev 2000;(2):CD002153. (Systematic review)
64. LoVecchio F, McBride C. Scorpion envenomations in
young children in central Arizona. J Toxicol Clin
Toxicol 2003;41(7):937-40. (Prospective case series; 483
patients)
65. Riley BD, LoVecchio F, Pizon AF. Lack of Scorpion
Antivenom Leads to Increased Pediatric ICU
Admissions. Ann Emerg Med 2006;47(4):398-399.
(Before-after cohort; about 35,000 patients)
66. Gupta V. Prazosin: a pharmacological antidote for
scorpion envenomation. J Trop Pediatr 2006;52(2):150151. (Prospective, before-after, quasi-experimental;36
(Case Report)
39. Baur X, Chen Z, Rozynek P, Düser D, et al. Cross-reacting IgE antibodies recognizing latex allergens, including Hev b 1, as well as papain. Allergy 1995;50(7):604609. (Clinical Trial, 58 patients)
40. Caravati EM, Copperhead bites and Crotalidae polyvalent immune Fab (ovine): Routine use requires evidence of improved outcomes. Ann Emerg Med
2004;43(2): 207-208. (Expert opinion)
41. Lavonas EJ, Gerardo CJ, O’Malley G, et al. Initial experience with Crotalidae polyvalent immune Fab (ovine)
antivenom in the treatment of copperhead snakebite.
Ann Emerg Med 2004;43:200-206. (Retrospective chart
review;32 patients)
42. Jurkovich GJ, Luterman A, McCullar K, et al.
Complications of Crotalidae antivenin therapy. J Trauma
1988;28(7):1032-1037. (Prospective, Cohort; 40 patients)
43. Schmidt JM. Antivenom therapy for snakebites in children: is there evidence? Curr Opin Pediatrics
2005;17(2):234-238. (Review)
44. Offerman SR, Bush SP, Moynihan JA, et al. Crotaline
Fab antivenom for the treatment of children with rattlesnake envenomation. Pediatrics 2002;110: 968–971.
(Prospective/Retrospective, pediatric cohort; 12
patients)
45. Dart RC, McNally J. Efficacy, safety, and use of snake
antivenoms in the United States. Ann Emerg Med
2001;37(2):181-188. (Review)
46. Holstege CP, Wu J, Baer AB. Immediate hypersensitivity reaction associated with the rapid infusion of
Crotalidae polyvalent immune Fab (ovine). Ann Emerg
Med 2002; 39(6):677-679. (Case report)
47. Clark RF, McKinney PE, Chase PB. et al. Immediate
and delayed allergic reactions to Crotalidae polyvalent
immune Fab (ovine) antivenom. Ann Emerg Med
2002;39(6):671-676.
48. Boyer LV, Seifert SA, Clark RF, et al. Recurrent and
persistent coagulopathy following pit viper envenomation. Arch Intern Med 1999;159(7):706-710.
(Retrospective chart review; 38 patients)
49. Boyer LV, Seifert SA, Cain JS. Recurrence phenomena
after immunoglobulin therapy for snake envenomations: part 2. Guidelines for clinical management with
Crotaline Fab antivenom. Ann Emerg Med
2001;37(2):196-201. (Review and clinical guidelines)
50. Holstege CP, Miller MB, Wermuth M, et al. Crotalid
snake envenomation. Crit Care Clin 1997;13(4):889-921.
(Review Article)
51. Stewart RM, Page CP, Schwesinger WH, et al.
Antivenin and fasciotomy/debridement in the treatment of the severe rattlesnake bite. Am J Surg
1989;158:543-547. (Animal Study, randomized, controlled)
52. Tanen DA, Danish DC, Grice GA, et al. Fasciotomy
worsens the amount of myonecrosis in a porcine
model of Crotaline envenomation. Ann Emerg Med
2004;44(2):99-104. (Randomized, blinded, controlled
animal; 20 pigs)
Emergency Medicine Practice©
24
September 2006 • EBMedice.net
d. Grade III/IV (severe/very severe envenomation)
patients)
67. McClain C. 2-year scorpion-antivenin trials successful;
FDA scrutiny next. Accessed 6 July 2006 at
http://www.azstarnet.com/allheadlines/124591.
(Unpublished, prospective, randomized, controlled;
50 patients)
68. Hughes A. Observation of snakebite victims: is twelve
hours still necessary? Emerg Med (Freemantle)
2003;15(5-6):511-517. (Two phase case series; 360
patients)
69. Norris R. “Snake Venom Poisoning In The United
States: A Medical Emergency!” Accessed 11 September
2006 at http://www.emed.stanford.education/didactics/snakebites.htm.
70. Gibly R, Williams M, Walter FG et al. Continuous
intravenous midazolam infusion for Centruroides exilicauda scorpion envenomation. Ann Emerg Med
1999;34(5): 669-670.
36. If the patient in question three developed no
further symptoms, what is the correct management plan?
a. Mix CroFabTM and administer four vials
according to the treatment algorithm and
then discharge home.
b. Mix CroFabTM and administer four vials
according to the treatment algorithm and
then admit for the completion of the
CroFabTM treatment regimen.
c. Observe for four hours and discharge if
unchanged.
d. Observe for eight hours and discharge if
unchanged.
CME Questions
37. If the patient in question three developed significant altered mental status, but no other
changes in his presentation, what grade should
be assigned to his envenomation?
33. Which of the following anatomic regions is the
location for more than 95% of snake envenomations?
a. Back
b. Extremities
c. Head
d. Torso
a. Grade 0 (nonenvenomation)
b. Grade I (mild envenomation)
c. Grade II (moderate envenomation)
d. Grade III/IV (severe/very severe envenomation)
34. Which of the following is a true statement?
38. An 8-year-old, 35kg female presents to the
emergency department one hour after an identified cottonmouth/water moccasin bite to the
leg while swimming in a local pond. She has
marked pain, severe edema and erythema to
her entire leg, and has an elevated INR and
aPTT. Appropriate management should
include which of the following?
a. 25% of venomous snakebites in the US are
fatal.
b. 98% of venomous snakebites in the US are
from Crotalids.
c. Imported snakes account for the majority of
fatal snakebites in the US.
d. Up to 20% of the venomous snakebites in the
US can be attributed to coral snakes.
a. Three vials of CroFabTM, adjusted for weight,
mixed in 250cc NS, and given according to
the treatment protocol.
b. Six vials of CroFabTM, with no weight adjustment, mixed in 250cc NS and given according to the treatment protocol.
c. Pretreatment with epinephrine 0.25mg IM
prior to CroFabTM administration.
d. Skin testing to determine sensitivity to
CroFabTM.
35. A 25-year-old male presents to the ED 60 minutes after a brown snake bit his hand. He says
the snake had a triangular head and a rattle on
its tail. Currently, the patient has local
swelling around 2 fang marks, some mild pain
in his hand, and minimal local swelling. His
vitals and labs are normal. What Grade is this
envenomation?
a. Grade 0 (nonenvenomation)
b. Grade I (mild envenomation)
c. Grade II (moderate envenomation)
EBMedicine.net • September 2006
39. All of the following patients have received a
25
Emergency Medicine Practice©
surgery for fasciotomy if greater than
40mmHg.
d. Measure compartment pressures and administer an additional four to six vials of
CroFabTM with mannitol and arm elevation if
pressures are greater than 40mmHg.
dose of four to six vials of CroFabTM. Which of
the following has achieved “initial control”?
a. Swelling and redness have improved according to the pen marks, and repeat labs show
an increase in INR from 2.5 to 3.0.
b. Swelling and redness have progressed
beyond the pen marks and the repeat labs
have returned to baseline normal levels.
c. Swelling and redness have remained the
same according to the pen marks and the
repeat labs are improved.
d. Swelling and redness have remained the
same according to the pen marks, the repeat
labs are improved, and the patient has developed confusion.
42. Which of the following is a true statement?
a. Mojave rattlesnake envenomations present in
a similar fashion to other Crotalid envenomations and can be assessed using the grading scale in Table 1.
b. Coral snake venom is classically considered
to be primarily cardiotoxic.
c. Crotalid venom is a complex mixture of proteins and other substances that affect the cardiac, neurologic, hematologic, and musculoskeletal systems.
d. Venom extraction devices are an effective
field treatment for envenomations and
should be applied by bystanders or EMS
providers that have them available in the
field.
40. The patient has received six vials of CroFabTM
and has achieved initial control of the envenomation. Three hours after his first two vial follow-up dose of CroFabTM, his altered mental
status returns. What is the best course of
action at this point?
a. Administer two vials of CroFabTM and continue with his scheduled dosing regimen
according to the original times.
b. Administer two vials of CroFabTM and reset
the follow vial timing for additional doses at
6, 12, and 18 hours following the newest
dose.
c. Administer four to six vials of CroFabTM and
reassess in one hour.
d. Wait until the next six hour incremental dose
is due and administer two vials.
43. An 18-month-old, 12kg female was stung on the
leg by a scorpion while playing on her porch in
Tuscon, Arizona. Over the past few minutes,
she has started getting agitated, sweating profusely, vomiting, and passing watery stools. On
arrival to the ED by EMS, the patient was alert,
agitated, diaphoretic, tachypneic at 48 breaths
per minute with oxygen saturation 94% by
pulse oximetry, and tachycardic at 160 beats per
minute. After starting an IV line, placing oxygen by mask, and instituting continuous monitoring, your best initial course of action is to:
41. A patient who has received one dose of four
vials of CroFabTM following a severe (Grade III)
envenomation of the hand has developed
increased pain, pallor, and numbness of his
hand. Palpation of his forearm reveals a
very tight compartment and he has pain out
of proportion. What is the best course of
action?
a. Administer three vials scorpion antivenom
intravenously immediately with a 20ml/kg
bolus of normal saline.
b. Give 0.12mg epinephrine 1:1000 intramuscularly in the thigh and 12.5mg diphenhydramine intravenously.
c. Give midazolam intravenously, draw labs,
and plan for pediatric intensive care unit
admission.
d. Give metoprolol 2.5mg intravenously every
five minutes for three doses to establish betareceptor blockade, but hold for heart rate
less than 60 beats per minute.
a. Continue CroFabTM treatment according to
timed protocol, elevate his arm, and reexamine in one hour.
b. Immediate fasciotomy to relieve compartment sydrome.
c. Measure compartment pressures and consult
Emergency Medicine Practice©
26
September 2006 • EBMedice.net
the patient is anxious, tachypneic at 32 breaths
per minute, with faint wheezes bilaterally.
After establishing IV access, placing oxygen by
nasal cannula, and instituting continuous cardiac monitoring, what is the best initial course
of action?
44. The patient is a 39-year-old man who was bitten
on the hand while clearing brush at his Florida
home by a red, yellow, and black snake,
thought to be a coral snake. Injury occurred 30
minutes before ED arrival and the patient has
no symptoms, no pain, no paresthesias, and no
neurological deficits. What is the best course
of action?
a. Give 0.3mg epinephrine 1:1000 intramuscularly in the thigh and 50mg diphenhydramine intravenously.
b. Give three more vials of coral snake antivenom intravenously for progression of symptoms.
c. Perform neuro checks every hour to assess
for progression of symptoms.
d. Perform rapid sequence intubation for
impending respiratory failure.
a. Observe for 12 hours and, if no symptoms,
discharge home.
b. Obtain CBC, CMP, PT/PTT and fibrinogen at
baseline and after four hours while monitoring for symptoms.
c. Immediately give coral snake antivenom.
d. Perform neuro checks every hour and give
antivenom when and if symptoms appear.
47. The patient is a 44-year-old man who received
polyvalent, horse serum-derived antivenom for
bites from a Gaboon viper while working at
the zoo two weeks ago. He presents to the ED
for evaluation of an itchy, red rash on trunk
and arms that has been increasing for the past
three days. What is the best treatment plan for
this patient?
45. The patient is a 26-year-old man who was cleaning up in the trailer of a circus snake handler
and decided to “mess” with the snakes. A
monocle cobra bit him at least twice and
maybe three times on the right hand and forearm about two hours ago. He tried to conceal
his injury from his boss, but started to have
severe pain, muscle twitching, and difficulty
swallowing so admitted the injury and was
brought into the ED. The snake handler brings
ten vials of polyvalent cobra antivenom with
the patient. What is the best initial course of
action?
a. Obtain additional Gaboon viper antivenom
to give additional treatment for unresolved
envenomation.
b. Give 0.3mg epinephrine 1:1000 intramuscularly in the thigh and 50mg diphenhydramine intravenously.
c. Obtain CBC, CMP, PT/PTT and fibrinogen at
baseline and after four hours while monitoring for symptoms.
d. Give prednisone 60mg and diphenhydramine 50mg PO in the ED and prescribe a
steroid taper for ten days and prn diphenhydramine.
a. Call the poison control and the regional zoo
for guidance in managing this patient.
b. Give midazolam 4mg intravenously, draw
labs, and plan for ICU admission.
c. Give 0.25mg epinephrine subcutaneously to
prevent immediate hypersensitivity reaction
and three vials of cobra venom intravenously.
d. Obtain CBC, CMP, PT/PTT, and fibrinogen
at baseline and perform type and cross for
packed red blood cells and fresh frozen plasma.
48. The patient is a 36-year-old collector of venomous snakes who was bitten by one of his
black mamba (Dendroaspis polylepis) snakes
about one hour ago. He has no clear symptoms,
but says he feels funny and anxious. His vital
signs are stable and he is in no distress. He
reports that he has antivenom for almost all of
his snakes, but he has never been able to obtain
antivenom for this snake species. After establishing IV access, placing oxygen by nasal can-
46. The patient is a 52-year-old woman who was
bitten by a coral snake two hours ago and
given three vials of coral snake antivenom
about ten minutes ago. She complains of an
intensely itchy, blotchy rash all over the body
and face and swelling of the lips. On exam,
EBMedicine.net • September 2006
27
Emergency Medicine Practice©
Committees of the American Heart
Association and representatives
from the resuscitation councils of
ILCOR: How to Develop EvidenceBased Guidelines for Emergency
Cardiac Care: Quality of Evidence
and Classes of Recommendations;
also: Anonymous. Guidelines for
cardiopulmonary resuscitation and
nula, and instituting continuous cardiac monitoring, what is the best initial course of action?
a. Call your local poison center or national poison center hotline 800-222-1222 for help
obtaining antivenom.
b. Give coral snake antivenom three vials intravenously to capitalize on cross-reactivity of
the species.
c. Give 0.3mg epinephrine 1:1000 intramuscularly in the thigh and 50mg diphenhydramine intravenously.
d. Obtain CBC, CMP, PT/PTT, and fibrinogen
at baseline and after four hours while monitoring for symptoms.
emergency cardiac care. Emergency
Cardiac Care Committee and
Subcommittees, American Heart
Association. Part IX. Ensuring effectiveness of community-wide emergency cardiac care. JAMA
1992;268(16):2289-2295.
Physician CME Information
Credit Designation: The Mount Sinai School of Medicine designates this
educational activity for a maximum of 48 AMA PRA Category 1
Credit(s)TM per year. Physicians should only claim credit commensurate
with the extent of their participation in the activity.
Credit may be obtained by reading each issue and completing the printed
post-tests administered in December and June or online single-issue
post-tests administered at EBMedicine.net.
Target Audience: This enduring material is designed for emergency medicine physicians.
Needs Assessment: The need for this educational activity was determined
by a survey of medical staff, including the editorial board of this publication; review of morbidity and mortality data from the CDC, AHA, NCHS,
and ACEP; and evaluation of prior activities for emergency physicians.
Date of Original Release: This issue of Emergency Medicine Practice was
published September 12, 2006. This activity is eligible for CME credit
through September 12, 2009. The latest review of this material was
September 1, 2006.
Coming in Future Issues:
Acutely Decompensated Heart Failure
Weakness
Complications In Pregnancy
Discussion of Investigational Information: As part of the newsletter, faculty may be presenting investigational information about pharmaceutical
products that is outside Food and Drug Administration approved labeling.
Information presented as part of this activity is intended solely as continuing medical education and is not intended to promote off-label use of
any pharmaceutical product. Disclosure of Off-Label Usage: This issue of
Emergency Medicine Practice discusses no off-label use of any pharmaceutical product.
Class Of Evidence Definitions
Each action in the clinical pathways section of Emergency Medicine
Practice receives a score based on the following definitions.
Class I
• Always acceptable, safe
• Definitely useful
• Proven in both efficacy and
effectiveness
Class III
• May be acceptable
• Possibly useful
• Considered optional or alternative
treatments
Level of Evidence:
• One or more large prospective
studies are present (with rare
exceptions)
• High-quality meta-analyses
• Study results consistently positive
and compelling
Level of Evidence:
• Generally lower or intermediate
levels of evidence
• Case series, animal studies, consensus panels
• Occasionally positive results
Class II
• Safe, acceptable
• Probably useful
Level of Evidence:
• Generally higher levels of evidence
• Non-randomized or retrospective
studies: historic, cohort, or casecontrol studies
• Less robust RCTs
• Results consistently positive
Faculty Disclosure: It is the policy of Mount Sinai School of Medicine to
ensure objectivity, balance, independence, transparency, and scientific
rigor in all CME-sponsored educational activities. All faculty participating
in the planning or implementation of a sponsored activity are expected to
disclose to the audience any relevant financial relationships and to assist
in resolving any conflict of interest that may arise from the relationship.
Presenters must also make a meaningful disclosure to the audience of
their discussions of unlabeled or unapproved drugs or devices.
In compliance with all ACCME Essentials, Standards, and Guidelines, all
faculty for this CME activity were asked to complete a full disclosure
statement. The information received is as follows: Dr. Costello, Dr. Heins,
Dr. Zirkin, Dr. Lovecchio, Dr. Barish, and Dr. Stewart report no significant
financial interest or other relationship with the manufacturer(s) of any
commercial product(s) discussed in this educational presentation.
For further information, please see The Mount Sinai School of Medicine
website at www.mssm.edu/cme.
Indeterminate
• Continuing area of research
• No recommendations until further
research
ACEP Accreditation: Emergency Medicine Practice is approved by the
American College of Emergency Physicians for 48 hours of ACEP
Category 1 credit per annual subscription.
Level of Evidence:
• Evidence not available
• Higher studies in progress
• Results inconsistent, contradictory
• Results not compelling
AAFP Accreditation: Emergency Medicine Practice has been reviewed
and is acceptable for up to 48 Prescribed credits per year by the
American Academy of Family Physicians. AAFP Accreditation begins
August 1, 2006. Term of approval is for two years from this date. Each
issue is approved for 4 Prescribed credits. Credits may be claimed for
two years from the date of this issue.
Significantly modified from: The
Emergency Cardiovascular Care
AOA Accreditation: Emergency Medicine Practice has been approved for
48 Category 2B credit hours per year by the American Osteopathic
Association.
Emergency Medicine Practice is not affiliated with any pharmaceutical firm or medical device manufacturer.
CEO: Robert Williford President and Publisher: Stephanie Williford
Direct all editorial or subscription-related questions to EB Medicine: 1-800-249-5770 • Fax: 1-770-500-1316 • Non-U.S. subscribers, call: 1-678-366-7933
EB Practice, LLC • 305 Windlake Court • Alpharetta, GA 30022
E-mail: [email protected] • Web Site: EBMedicine.net
Emergency Medicine Practice (ISSN Print: 1524-1971, ISSN Online: 1559-3908) is published monthly (12 times per year) by EB Practice, LLC, 305 Windlake Court, Alpharetta, GA 30022. Opinions
expressed are not necessarily those of this publication. Mention of products or services does not constitute endorsement. This publication is intended as a general guide and is intended to supplement, rather than substitute, professional judgment. It covers a highly technical and complex subject and should not be used for making specific medical decisions. The materials contained herein
are not intended to establish policy, procedure, or standard of care. Emergency Medicine Practice is a trademark of EB Practice, LLC. Copyright © 2006 EB Practice, LLC. All rights reserved. No
part of this publication may be reproduced in any format without written consent of EB Practice, LLC. Subscription price: $299, U.S. funds. (Call for international shipping prices.)
Emergency Medicine Practice©
28
September 2006 • EBMedice.net